Đến nội dung

Hình ảnh

Đề thi chọn HSG lớp 12 tỉnh Bình Định năm học 2014-2015


  • Please log in to reply
Chủ đề này có 5 trả lời

#1
LNH

LNH

    Bất Thế Tà Vương

  • Hiệp sỹ
  • 581 Bài viết

   SỞ GIÁO DỤC VÀ ĐÀO TẠO                                             KỲ THI CHỌN HSG TỈNH LỚP 12 THPT

                 BÌNH ĐỊNH                                                                       KHÓA NGÀY:22-10-2014

 

            ĐỀ CHÍNH THỨC                                Môn thi:               Toán

                                                                          Thời gian:             180 phút

                                                                          Ngày thi:              22/10/2014

                                                                          -----------------------------------------------

Bài 1: (4 điểm )

Giải hệ phương trình:

$\left\{\begin{matrix} 2x+\frac{1}{x+y}=3\\ 4xy+4x^2+4y^2+\frac{3}{\left ( x+y \right )^2}=7 \end{matrix}\right.$

Bài 2: (4 điểm)

a)      Cho p là một số nguyên tố, k là một số nguyên dương. Một đường trong được chia thành p cung bằng nhau. Tiến hành tô các cung bằng k màu khác nhau ( mỗi cung được tô bằng một màu). Hai cách tô màu được coi là giống nhau nếu cách tô này sẽ thu được từ cách tô kia qua một phép quay với tâm là tâm của đường tròn. Hỏi có bao nhiêu cách tô màu khác nhau? (Cách chứng minh định lí Fermat nhỏ bằng tổ hợp)

b)      Tìm tất cả các đa thức  thỏa mãn điều kiện: $P\left( x \right)=\sqrt{P\left( {{x}^{2}}+1 \right)-7}+6,\forall x\ge 0,P\left( 0 \right)=6$

Bài 3: (4 điểm)

Cho số thực . Xét dãy số  xác định bởi:

$\left\{\begin{matrix} x_1=a\\ x_{n+1}=1+ln\left ( \frac{x_n^2}{1+lnx_n} \right ) \end{matrix}\right.$ với n=1,2,…

Chứng minh rằng dãy số có giới hạn hữu hạn và tìm giới hạn đó.

Bài 4: (4 điểm)

Cho tam giác ABC nội tiếp đường tròn (O;R). Với mỗi điểm M trong đường tròn ta gọi A’,B’,C’ lần lượt là giao điểm của AM,BM,CM với đường tròn. Tìm tập hợp các điểm M trong đường tròn thỏa mãn hệ thức sau:

$\frac{MA}{MA'}+\frac{MB}{MB'}+\frac{MC}{MC'}\le 3$

Bài 5: (4 điểm)

Cho 2 điểm cố định $A, B$ và điểm  di động trên mặt phẳng sao cho $\hat{ACB}=a \ (0<a<180)$ không đổi cho trước. Hình chiếu của tâm đường tròn nội tiếp $I$ của tam giác $ABC$ xuống ba cạnh $AB,\ BC,\ CA$ lần lượt là $D,E,F$. $AI$ và $BI$ cắt $EF$ lần lượt tại $M,N$.
a) Chứng minh độ dài $MN$ không đổi.
b) CM đường tròn $(DMN)$ luôn đi qua một điểm cố định.


Bài viết đã được chỉnh sửa nội dung bởi LNH: 22-10-2014 - 12:59


#2
Near Ryuzaki

Near Ryuzaki

    $\mathbb{NKT}$

  • Thành viên
  • 804 Bài viết

   SỞ GIÁO DỤC VÀ ĐÀO TẠO                                             KỲ THI CHỌN HSG TỈNH LỚP 12 THPT

                 BÌNH ĐỊNH                                                                       KHÓA NGÀY:22-10-2014

 

            ĐỀ CHÍNH THỨC                                Môn thi:               Toán

                                                                          Thời gian:             180 phút

                                                                          Ngày thi:              22/10/2014

                                                                          -----------------------------------------------

Bài 1: (4 điểm )

Giải hệ phương trình:

$\left\{\begin{matrix} 2x+\frac{1}{x+y}=3\\ 4xy+4x^2+4y^2+\frac{3}{\left ( x+y \right )^2}=7 \end{matrix}\right.$

 

Bài hệ :

$$\left\{\begin{matrix} 2x+\frac{1}{x+y}=3\\ 4xy+4x^2+4y^2+\frac{3}{\left ( x+y \right )^2}=7 \end{matrix}\right.\Rightarrow \left\{\begin{matrix} (x-y)+(x+y)+\frac{1}{x+y}=3\\3(x+y)^2+(x-y)^2+\frac{3}{(x+y)^2}=7 \end{matrix}\right.$$

Đặt $x+y=a,x-y=b$

ta được :

$$\left\{\begin{matrix} b+a+\dfrac{1}{a}=3\\3a^2+b^2+\dfrac{3}{a^2} =7 \end{matrix}\right.\Rightarrow \left\{\begin{matrix} b+(a+\frac{1}{a})=3\\3(a+\frac{1}{a})^2+b^2=13 \end{matrix}\right.$$

Từ đó tìm được $b$ ( thế) :))

Spoiler


Bài viết đã được chỉnh sửa nội dung bởi sieusieu90: 22-10-2014 - 13:11


#3
HungNT

HungNT

    Thượng sĩ

  • Thành viên
  • 273 Bài viết

 

Bài 5: (4 điểm)

Cho 2 điểm cố định $A, B$ và điểm  di động trên mặt phẳng sao cho $\hat{ACB}=a \ (0<a<180)$ không đổi cho trước. Hình chiếu của tâm đường tròn nội tiếp $I$ của tam giác $ABC$ xuống ba cạnh $AB,\ BC,\ CA$ lần lượt là $D,E,F$. $AI$ và $BI$ cắt $EF$ lần lượt tại $M,N$.
a) Chứng minh độ dài $MN$ không đổi.
b) CM đường tròn $(DMN)$ luôn đi qua một điểm cố định.

chém câu nào dế nhất

5.

untitled.PNG

a.Nếu E nằm giữa N,F thì $\angle AEN=\angle CEF=90^{0}-\frac{\alpha}{2}=\frac{\angle A+\angle B}{2}$

hay $\angle AEN=\angle AIN$

Nếu N nằm giữa E,F thì $\angle AEN+\angle AIN=180^{0}$

Suy ra tứ giác AIEN nội tiếp$=>\angle MNI=\angle IAB=>\Delta MNI\sim \Delta BAI$

$=>MN=\frac{NI}{AI}.AB=cos(90^{0}-\frac{\alpha }{2}).AB=const$

b.MB cắt AN tại K, do AM,BN,KD là các đường cao của $\Delta KAB$

Suy ra đường tròn ngoại tiếp $\Delta DMN$ là đường tròn Euler của $\Delta KAB$ đi qua trung điểm AB


Bài viết đã được chỉnh sửa nội dung bởi HungNT: 23-10-2014 - 06:22


#4
phuocdinh1999

phuocdinh1999

    Thượng sĩ

  • Thành viên
  • 265 Bài viết

  Bài 2: (4 điểm)

a)      Cho p là một số nguyên tố, k là một số nguyên dương. Một đường trong được chia thành p cung bằng nhau. Tiến hành tô các cung bằng k màu khác nhau ( mỗi cung được tô bằng một màu). Hai cách tô màu được coi là giống nhau nếu cách tô này sẽ thu được từ cách tô kia qua một phép quay với tâm là tâm của đường tròn. Hỏi có bao nhiêu cách tô màu khác nhau? (Cách chứng minh định lí Fermat nhỏ bằng tổ hợp)

b)      Tìm tất cả các đa thức  thỏa mãn điều kiện: $P\left( x \right)=\sqrt{P\left( {{x}^{2}}+1 \right)-7}+6,\forall x\ge 0,P\left( 0 \right)=6$

 

b) Xét dãy số $(a_n):$ $a_0=6$ ; $a_(n+1)=a_n^2+1$

Dễ cm dãy trên tăng không bị chặn $(1)$

- Xét $Q(x)=P(x)-(x+6)$ , từ đó: $Q(x)+x=\sqrt{Q(x^2+1)+x^2}\Rightarrow Q(a_n)+a_n=\sqrt{Q(a_n^2+1)+a_n^2}$

Hay $Q(a_n)+a_n=\sqrt{Q(a_(n+1)+a_n^2}$

Ta có $Q(a_0)=Q(6)=0$, bằng quy nạp ta cm được $Q(a_n)=0\, \, \forall n\in \mathbb{N}*$

Kết hợp với $(1)$ suy ra $Q(x)$ có vô số nghiệm $\Rightarrow Q(x)\equiv 0\Rightarrow P(x)\equiv x+6$



#5
hoangtubatu955

hoangtubatu955

    Sĩ quan

  • Thành viên
  • 429 Bài viết

ScreenHunter_16%20Oct.%2022%2020.30.jpg?

Gọi $ K $ là giao điểm của $ EF $ và $ AB $.
Ta có: $ (KD,AB)=-1) $.
Từ đây do $ MA $  và $ NB $ là phân giác của các góc $ KMD $ và $ DNB $ nên $ ANB=AMB=90^o $.
Từ đây thì tứ giác $ ANMB $ nội tiếp đường tròn đường kính $ AB $ tâm $ J $. Kết hợp $ MJN=ACB $ không đổi nên $ MN $ không đổi.
Câu b.
Theo hệ thức Marclaurin thì $ KD.KJ=KA.KB=KN.KM $ nên $ (MND) $ luôn đi qua $ J $ cố định. $ Q.E.D $

Bài viết đã được chỉnh sửa nội dung bởi hoangtubatu955: 22-10-2014 - 20:55


#6
lenhatsinh3

lenhatsinh3

    Hạ sĩ

  • Thành viên
  • 86 Bài viết

Bài 4: $\frac{MA}{M'A}+\frac{MB}{M'B}+\frac{MC}{M'C}\leq 3\Leftrightarrow \frac{MA^{2}}{MA'.MA}+\frac{MB^{2}}{MB'.MB}+\frac{MC^{2}}{MC'.MC}\leq 3$

$\Leftrightarrow \frac{MA^{2}+MB^{2}+MC^{2}}{R^{2}-MO^2}\leq 3$$\left ( 1 \right )$

Gọi $G$ là trọng tâm $\Delta ABC$ 

$MA^{2}+MB^{2}+MC^{2}=3MG^{2}+GA^{2}+GB^{2}+GC^{2}$

$3R^{2}-3MO^{2}=OA^{2}+OB^{2}+OC^{2}-3MO^{2}=3OG^{2}+GA^{2}+GB^{2}+GC^{2}-3MO^{2}$

$\left ( 1 \right )\Leftrightarrow MA^{2}+MB^{2}+MC^{2}\leq 3R^{2}-MO^{2}\Leftrightarrow MO^{2}+MG^{2}\leq OG^{2}$

Diều này chứng tỏ $M$ nằm trong hình  tròn đường kính $OG$ tâm là trung điểm $OG$


:ukliam2:  :ukliam2:  :ukliam2:  :ukliam2:  :ukliam2:

      :ukliam2:

            :ukliam2:

                  :ukliam2:

             :ukliam2:

        :ukliam2:  

     :ukliam2:  :ukliam2:  :ukliam2:  :ukliam2:  :ukliam2:





1 người đang xem chủ đề

0 thành viên, 1 khách, 0 thành viên ẩn danh